Tải bản đầy đủ (.pdf) (16 trang)

graph 2017 kon tum

Bạn đang xem bản rút gọn của tài liệu. Xem và tải ngay bản đầy đủ của tài liệu tại đây (330.06 KB, 16 trang )

BẬC CỦA MỘT ĐỈNH TRONG ĐỒ THỊ
Võ Quang Vinh, trường Chuyên Nguyễn Tất Thành Kon Tum
Email: ; ĐT: 0935039007
Trong bài viết này, tôi giới thiệu một số kiến thức về lý thuyết đồ thị và một lớp
các bài toán liên quan đến việc đánh giá bậc của các đỉnh trong đồ thị để giải quyết.
I. Một số kiến thức về đồ thị.
1.1. Định nghĩa đồ thị.
- Một đồ thị vô hướng là một cặp G  (V , E ) trong đó V là tập hữu hạn các đỉnh
và E  V  V là tập hữu hạn các cặp đỉnh (không có thứ tự) – gọi là tập các cạnh.
- Với A, B V , nếu ( A, B)  E thì A, B được gọi là hai đỉnh kề nhau. Cạnh ( A, B)
và đỉnh A (hay đỉnh B ) được gọi là liên thuộc với nhau.
- Tập các đỉnh kề với A ký hiệu là N  A .
- Một cạnh nối một đỉnh với chính nó gọi là 1 khuyên.
- Đồ thị đơn là đồ thị không có khuyên và giữa hai đỉnh có nhiều nhất một cạnh.
Nếu không có ghi chú đặc biệt thì trong phần còn lại của bài viết ta ngầm định đồ thị
là đồ thị đơn.
1.2. Đồ thị con, đồ thị cảm sinh, đồ thị bù.
Cho đồ thị G  (V , E ) .
- Đồ thị G '  (V ', E ') thoả mãn V '  V ; E '  E được gọi là một đồ thị con của
G  (V , E ) .
- G '  (V ', E ') một đồ thị con của G  (V , E ) và với mọi
A, B  V ' ( A, B )  E '  ( A, B)  E thì G '  (V ', E ') gọi là đồ thị cảm sinh của đồ thị
G  (V , E ) lên tập V ' . Ký hiệu là G[V'] .
- Đồ thị H  (V , VxV\ E ) được gọi là đồ thị bù của đồ thị G  (V , E ) .
1.3. Bậc của đỉnh đồ thị.
Cho đồ thị G  (V , E ) và A  V .
- Số đỉnh kề với A được gọi là bậc của A trong G  (V , E ) kí hiệu
deg G A | N ( A) | .
- Một đỉnh có bậc 0 gọi là đỉnh cô lập.
Định lí 1. Cho đồ thị đơn vô hướng G  (V , E ) .
i)  degG A  2 | E | .


AV

ii) Số đỉnh có bậc lẻ luôn là số chẵn.
Định lí 2. Cho đồ thị G  (V , E ) và V '  V . Với mọi A V ' ta có
deg G A  degG[V '] A  degG[ V \V {A}] A .
 


Định lí 3. Cho đồ thị đơn vô hướng G  (V , E ) . Khi đó
1



( A; B )E

(deg( A)  deg( B))   (deg A) 2
AV

1.4. Đường đi, chu trình, đồ thị liên thông.
Cho đồ thị G  (V , E ) và A, B V .
- Một đường đi từ A đến B là một dãy các cạnh ( Ai , Ai 1 ), i  0,.., k  1 với
A0  A; Ak  B .
- Số các cạnh của đường đi gọi là độ dài của đường đi. Ta chỉ quan tâm đến
những đường đi có độ dài lớn hơn 1 và không sử dụng cạnh nào quá một lần hay
thậm chí không sử dụng một đỉnh nào quá 1 lần.
- Nếu ta có 1 đường đi từ A đến B và ( A, B)  E thì ta gọi đó là 1 chu trình.
Định lí 4. Nếu một đồ thì có bậc của mỗi đỉnh đều lớn hơn 1 thì đồ thị cđó chứa
một chu trình.
II. Một số bài toán liên quan đến bậc của đỉnh của đồ thị.
Với ý tưởng sử dụng đồ thị để giải các bài toán về tổ hợp nên trong các bài toán

được trình tiếp theo có thể có những lời giải đơn giản hơn, hay hơn lời giải được
trình bày.
Bài toán 1: Cho 6 điểm phân biệt nằm trên 1 đường tròn, nối 2 điểm bất kì bởi
một đường màu đỏ hoặc màu xanh. Chứng minh rằng:
a) Tồn tại ít nhất một tam giác có 3 cạnh cùng màu.
b) Tồn tại ít nhất hai tam giác có 3 cạnh cùng màu.
Phân tích và lời giải.
Ta coi mỗi điểm đã cho là một đỉnh của đồ thị. Nếu 2 điểm được nối với nhau
bằng đường đỏ ta gọi đó là 1 cạnh. Nếu 2 điểm nối với nhau bằng đường xanh ta coi
đó không phải là 2 đỉnh kề. Ta thu được một đồ thị G  (V , E ) .
Yêu cầu bài toán đặt ra là chứng minh rằng tồn tại một chu trình có độ dài bằng
3 hoặc tồn tại 3 đỉnh mà đồ thị cảm sinh trên tập gồm 3 điểm đó không chứa cạnh.
a) Lấy đỉnh A V . Ta phân hoạch tập đỉnh còn lại của đồ thị thành 2 tập, tập
N ( A)  {B  V | (A, B)  E} và M ( A)  {B  V | (A, B)  E} .
Khi đó | N ( A) |  | M(A) | 5 theo nguyên lý Dirichlet tồn tại 1 tập có bản số lớn
hơn hoặc bằng 3.
Trường hợp 1: degG A  3 lấy 3 đỉnh A1; A2 ; A3  N ( A) .

+ Nếu tồn tại ( Ai ; Aj )  E thì ( A; Ai ; Aj ) là tập các điểm cần tìm.
2


+ Nếu ngược lại thì ( A1; A2 ; A3 ) là tập các điểm cần tìm.
Trường hợp 2: degG A  3 ta có | M ( A) | 3 lấy B1; B2 ; B3  M ( A) .

+ Nếu tồn tại (Bi ; B j )  E thì ( A; Bi ; B j ) là tập các điểm cần tìm.
+ Nếu ngược lại thì ( B1; B2 ; B3 ) là tập các điểm cần tìm.
b) Ta xét 3 trường hợp
Trường hợp 1: nếu trong G tồn tại đỉnh có bậc lớn hơn hoặc bằng 4, khi đó lý
luận tương tự trường hợp 1 câu a) ta suy ra có 2 tam giác khác nhau thoả điều

kiện đề bài.
Trường hợp 2: nếu trong G tồn tại đỉnh có bậc bé hơn hoặc bằng 4, khi đó lý
luận tương tự trường hợp 2 câu a) ta suy ra có 2 tam giác khác nhau thoả điều
kiện đề bài.
Trường hợp 3: mọi đỉnh của G đều có bậc bằng 3. Theo câu a) tồn tại 1 tam
giác thoả điều kiện đề bài. Giả sử đó là ( A1 ; A2 ; A3 ) có cách cạnh thuộc E.
 Khi đó nếu tồn tại một tam giác khác có 2 trong ba đỉnh trên thì bài toán
được giải quyết.
 Nếu ngược lại ta giả sử N ( A1 )  {A2 ; A3 ; A 4 }
 A2 ; A3  N ( A4 )
Xét đỉnh A4 , ta có  A1  N ( A4 )
degA  3
4


tương tự trường hợp 1 của câu a) ta suy ra tồn tại một tam giác. Tam giác này
thoả điều kiện đề bài và khác với tam giác ( A1 ; A2 ; A3 )
Chú ý: việc sử dụng đồ thị vào giải quyết bài toán có vẻ làm phức tạp lời giải,
tuy nhiên đây là một ví dụ đơn giản giúp làm quen với đồ thị. Câu a của bài toán có
thể giải quyết bằng nguyên lý Dirichlet, câu b có thể sử dụng bất đẳng thức đánh giá
số góc có 2 cạnh cùng màu để giải.
Bài toán 2: Cho bảng 2018x2018 ô vuông, tô màu tuỳ ý một số ô vuông của
bảng đó. Chứng minh rằng ta có thể tô màu thêm không quá 4035 ô trong các ô còn
lại sao cho số ô được tô màu ở mỗi hàng cũng như mỗi cột là số chẵn.
3


Phân tích và lời giải.
Nhận xét rằng vì số ô trong mỗi hàng cũng như mỗi cột là số chẵn nên số ô tô
màu chẵn khi và chỉ khi số ô không tô màu chẵn.

Ta xây dựng đồ thị G  (V ; E ) trong đó mỗi hàng hoặc mỗi cột là một đỉnh là
một đỉnh, nếu tại ô là giao của một hàng và một cột không được tô màu ta có một
cạnh của đồ thị. Bài toán được đưa về việc chứng minh có thể bỏ đi không quá 4035
cạnh để thu được một đồ thị mới mà bậc của mỗi đỉnh đều là số chẵn.
Vì không có thông tin về bậc của đỉnh nên để tạo ra thêm giả thiết ta thay
2018+2018 bởi n và chứng minh bằng phương pháp quy nạp. Trước hết, ta chứng
minh bổ đề:
Bổ đề: từ một đồ thị chứa n đỉnh có thể bỏ đi không quá n  1 cạnh để thu được
đồ thị mới có bậc của mỗi đỉnh đều là số chẵn.
Chứng minh.
Ta chứng minh bằng phương pháp quy nạp theo số đỉnh n của đồ thị.
Thật vậy, kết quả trên hiển nhiên đúng khi n  2 . Giả sử nó đúng với n  k ta
chứng minh đúng khi n  k  1 .
Xét đồ thị Gk 1 có k  1 đỉnh.
Trường hợp 1: trong Gk 1 tồn tại đỉnh cô lập A thì ta bỏ đỉnh A thu được đồ thị
mới có k đỉnh, áp dụng giả thiết quy nạp suy ra kết luận bài toán.

Trường hợp 2: trong Gk 1 tồn tại đỉnh A thoả degG A  1 thì ta bỏ đỉnh A và
cạnh liên kết với nó thu được đồ thị mới có k đỉnh, áp dụng giả thiết quy nạp suy ra
kết luận bài toán.
k 1

Trường hợp 3: mọi đỉnh của Gk 1 đều có bậc lớn hơn hoặc bằng 2. Khi đó, trong
Gk 1 tồn tại một chu trình. Để ý rằng nếu bỏ đi 1 chu trình thì không làm thay đổi tính
chẵn lẻ của bậc các đỉnh của đồ thị.
4


Vì số cạnh của đồ thị là hữu hạn nên nó chứa hữu hạn chu trình, bỏ đi tất cả các
chu trình đó ta thu được đồ thị mới có k  1 đỉnh thoả điều kiện của trường hợp 1

hoặc 2. Sau khi áp dụng giả thiết quy nạp và bổ sung vào lại các chu trình đã bỏ đi ta
thu được đồ thị thoả điều kiện đề bài. Vậy bổ đề được chứng minh.
Áp dụng bổ đề với n  4036 ta thu được kết quả bài toán.
Bài toán 3: Trong một hội nghị có 2018 người tham gia. Chứng minh rằng có
thể tìm được hai người tại hội nghị sao cho họ có một số chẵn (có thể là 0) người
quen chung tại hội nghị.
Phân tích và lời giải.
Ta xây dựng đồ thị G  (V ; E ) trong đó mỗi người tham gia hội nghị là một đỉnh,
nếu hai người quen biết nhau ta có một cạnh của đồ thị. Bài toán được đưa về việc
chứng minh tồn tại hai đỉnh A; B ( A  B ) sao cho | N ( A)  N ( B ) | là một số chẵn (có
thể là 0). Chú ý rằng | V | 2018 .
Giả sử ngược lại. Lấy đỉnh A  V , với mỗi X  N ( A) ta có degG[N (A)] X lẻ vì
| N ( A)  N ( X ) | lẻ. Điều này đúng với mọi X nên suy ra | N ( A) | chẵn.

Do đó degG A chẵn. Vì A được chọn ngẫu nhiên nên suy ra G (V ; E ) chỉ gồm các
đỉnh bậc chẵn.
Vẫn xét đỉnh A như ban đầu , với mỗi X  N ( A) ta cũng có
degG X  1  degG[N ( A)] X  deg G[M ( A) {X }] X
 degG[M ( A){X }] X  degG X  1  deg G[N ( A)] X

Suy ra degG[M ( A){X }] X chẵn.
Mặt khác, với mọi Y  M ( A) ta có degG[N( A){Y}] Y | N ( A)  N (Y ) | lẻ

5


Do đó degG[M ( A)] Y  deg G Y  deg G[N ( A){Y }] Y lẻ. Nên | M (A) | lẻ.
Điều này mâu thuẫn với | V || N ( A) |  | M ( A) | 1  2018 . Từ đó ta có điều phải
chứng minh.
Bài toán 4: Cho G  (V , E ) là một đồ thị đơn vô hướng. Chứng minh rằng có thể

phân hoạch tập đỉnh của G  (V , E ) thành hai tập V1 ; V2 sao cho đồ thị cảm sinh trên
hai tập trên có các đỉnh đều có bậc chẵn.
Phân tích và lời giải.
Đây là bài toán mà ta không có thông tin về đồ thị nên để giải quyết bài toán ta
có thể sử dụng nguyên lý quy nạp nhằm giúp có thêm giả thiết để giải quyết bài toán.
Chứng minh quy nạp theo số đỉnh của đồ thị n .
Khi n  1 hiển nhiên ta có kết quả bài toán. Giả sử bài toán đúng với mọi đồ thị
có it hơn n đỉnh.
Xét đồ thị G  (V , E ) có n đỉnh.
Nếu tất cả các đỉnh của đồ thị đều có bậc chẵn thì ta chọn V1  V ;V2   bài toán
được giải quyết.
Nếu có đỉnh A0  V có bậc lẻ. Để ý rằng | N ( A0 ) | lẻ.

Nhận xét rằng sau khi tách đỉnh A, nếu sử dụng giải thiết quy nạp với đồ thị
gồm n-1 đỉnh ta sẽ phân hoạch tập đỉnh ra thành 2 tập sao cho đồ thị cảm sinh đều
6


có bậc chẵn. Hai tập thu được có tính chẵn lẻ khi giao với N(A) khác nhau. Do đó ta
phải thêm A vào tập giao với N(A) chẵn để đảm bảo bậc của A trong đồ thị cảm sinh
là chẵn. Tuy nhiên, khi đó bậc của các đỉnh khác trong đồ thị cảm sinh sẽ có bậc lẻ
để giải quyết vấn đề này ta phải sử dụng đồ thị bù trước khi áp dụng giả thiết quy
nạp.
Ta định nghĩa đồ thị H  (V \{A0 }, E ') có tập đỉnh gồm n  1 đỉnh từ G  (V , E ) như
sau:

+ A, B  N ( A0 ) ta có ( A; B)  E '  (A; B)  E .
+Nếu A N ( A0 ) hoặc B  N ( A0 ) thì ( A; B)  E '  (A; B)  E .
Khi đó, theo giả thiết quy nạp V \{A0 } được phân hoạch thành hai tập U1 ;U 2 sao
cho đồ thị cảm sinh trên hai tập trên có các đỉnh đều có bậc chẵn.

Không mất tổng quát ta giả sử | N ( A0 )  U1 | chẵn và | N ( A0 )  U 2 | lẻ. Ta đặt
V1  U1  { A0 }
ta chứng minh hai tập này thoả yêu cầu bài toán.

V2  U 2

Thật vậy, xét X  V1 ta có
+ Nếu X  A0 khi đó degG[V ] X  deg G[V ] A0 | N ( A0 )  U1 | chẵn.
+ Nếu X  N ( A0 ) ta có degG[V ] X  deg H[V ] X chẵn.
1

1

1

7

1


+ Nếu X  N ( A0 ) ta có:
deg G[V1 ] X  deg G[V1 \ N ( Ao )] X  deg G[V1  N( Ao ){X}] X
 1  deg H[V1 \ N ( Ao )] X  | N ( A0 )  U1 | 1  deg H[V1  N( Ao ){X}] X
| N ( A0 )  U1 | 2 deg H[V1  N( Ao ){X}] X  deg H[V1 )] X

là số chẵn.
Tương tự xét X  V2 ta có
+ Nếu X  N ( A0 ) ta có degG[V ] X  deg H[V ] X chẵn.
+ Nếu X  N ( A0 ) ta có:
1


1

degG[V 2] X  degG[V12 N ( Ao )] X  degG[V2  N( Ao ) {X}] X
 deg H[V1 \ N ( Ao )] X  | N ( A0 )  U1 | 1  deg H[V1  N( Ao ){X}] X
| N ( A0 )  U 2 | 1  2 deg H[V2  N( Ao ) {X}] X  deg H[V2 )] X

là số chẵn.
Kết quả bài toán cho ta một cách phân hoạch các đỉnh của đồ thị sao cho các đỉnh
trong đồ thị cảm sinh đều có bậc chẵn. Tuy nhiên ta vẫn có thể phân hoạch sao cho
tính chẵn lẻ của chúng khác nhau.
Bài toán 5: Cho G  (V , E ) là một đồ thị đơn vô hướng. Chứng minh rằng có thể
phân hoạch tập đỉnh của G  (V , E ) thành hai tập V1 ; V2 sao cho:
+ Các đỉnh của đồ thị G[V ] có bậc chẵn.
+ Các đỉnh của G[V 2] có bậc lẻ.
Phân tích và lời giải.
Để thay đổi tính chẵn lẻ của bậc các đỉnh ta chỉ cần bỏ bớt 1 đỉnh có cạnh nối
với chúng. Do đó nghĩ tới việc thêm vào 1 đỉnh mới được nối với tất cả các đỉnh còn
lại sau đó bỏ nó đi.
Lấy một điểm O mới ngoài đồ thị, nối nó với tất cả các đỉnh của đồ thị ta thu
được đồ thị mới T . Áp dụng kết quả bài toán số 4 ta phân hoạch được tập đỉnh của
T thành hai tập W1; W2 sao cho đồ thị cảm sinh trên hai tập trên có các đỉnh đều có
bậc chẵn. Giả sử O  W2 , đặt V1  W1 ; V2  W2 \{O} , khi đó V1;V2 thoả mãn điều kiện đề
bài.
Bài toán 6: Giả sử, ở mỗi nút giao thông trong thành phố tổ hợp có một bóng
đèn và một công tắc. Ấn vào một công tắc sẽ thay đổi trạng thái bóng đèn tại nút đó
và các bóng đèn ở các nút kề nó (hai nút được gọi là kề nhau nếu có một đường đi từ
nút này đến nút kia). Ban đầu tất cả các bóng đèn đều tắt, chứng minh rằng người ta
có thể bật sáng tất cả các bóng đèn.
Phân tích và lời giải.

Gọi S là tập các công tắc ta cần thay đổi trạng thái (1 lần). Ta coi mạng lưới
này là một đồ thị, khi đó ta cần chọn ra S sao cho mỗi đỉnh của S được nối với số
chẵn lần các đỉnh khác trong S , và mỗi đỉnh không thuộc S được nối với số lẻ lần
các đỉnh thuộc S .
Ta xây dựng đồ thị G  (V ; E ) trong đó nút giao thông là một đỉnh, nếu hai nút
giao thông kề nhau ta có một cạnh của đồ thị.
1

8


Lấy một điểm O mới ngoài đồ thị, nối nó với tất cả các đỉnh có bậc chẵn của đồ
thị G ta thu được đồ thị mới H .

Theo bài toán 4 ta phân hoạch được tập đỉnh của H thành V1; V2 sao cho đồ thị
cảm sinh trên hai tập trên có các đỉnh đều có bậc chẵn.
Không mất tổng quát giả sử O V1 , khi đó ta chọn S  V2 . Ta có:
+ Nếu X  S thì deg G[S ] X  deg H[S ] X chẵn hay X được nối với chẵn đỉnh thuộc
S .
+ Nếu X  S  {O} ta xét 2 trường hợp:
i) X được nối với O trong V1 thì degG X ;deg H [V ] X đều chẵn nên
degG[S {X }] X  deg G X  1  deg H (V ) X lẻ.
ii) X không được nối với O trong V1 thì degG X lẻ và deg H [V ] X chẵn nên
degG[S {X }] X  degG X  deg H (V ) X lẻ.
Khi đó ta chỉ cần bậc tất cả công tắc thuộc S một lần thì bài toán được giải
quyết.
1

1


1

1

9


Bài toán 7: (China TST 1987) Cho số nguyên dương n (n  2) , trong mặt phẳng
cho 2n điểm, nối chúng bởi n 2  1 đoạn thẳng. Chứng minh rằng:
a) Tồn tại ít nhất một tam giác.
b) Tồn tại hai tam giác có chung cạnh.
c) Tồn tại ít nhất n tam giác.
Phân tích và lời giải.
Ta xây dựng đồ thị Gn  (V ; E ) trong đó điểm là một đỉnh là một đỉnh, nếu tại ô
là giao của một hàng và một cột không được tô màu hai điểm được nối với nhau ta có
một cạnh của đồ thị.
Ta chứng minh quy nạp theo n .
Khi n  2 ta có đồ thị gồm 4 đỉnh và 5 cạnh. Có thể thu được từ đồ thị đầy đủ
gồm 4 đỉnh bằng cách bỏ đi một cạnh nào đó nên nó chứa 2 tam giác có chung cạnh.
Hay các kết luận a); b); c) của bài toán đều đúng.
a) Giả sử kết luận a) đúng với n  1 hay Gn 1 chứa một tam giác.
Xét đồ thị Gn  (V ; E ) , lấy hai đỉnh A; B V sao cho ( A; B)  E .
Nếu tồn tại C  N ( A)  N ( B) thì ta có tam giác ( A; B; C ).

Nếu N ( A)  N ( B)   , khi đó degG A  1  deg B B  1  2n  2 .

| VH | n  2

Xét đồ thị H  (VH ; EH )  G[V \{A;B}] có 


2
2
| EH | n  1  (2 n  2  1)  (n  1)  1

10


do đó, theo giả thiết quy nạp thì H chứa một tam giác. Từ đó suy ra Gn  (V ; E )
luôn chứa ít nhất một tam giác.
b) Giả sử kết luận b) đúng với n  1 hay Gn 1 chứa 2 tam giác có chung cạnh.
Xét đồ thị Gn  (V ; E ) , theo kết quả a) tồn tại 1 tam giác. Ta giả sử
Gn  (V ; E ) chứa tam giác ( A; B; C ) và degG A  deg B B  degG C .
Nếu có D  ( N ( A)  N ( B))  ( N (B)  N (C))  ( N (C)  N (A)) thì bộ 4 đỉnh
( A; B; C ; D ) tạo thành tam giác 2 tam giác có chung cạnh.

Nếu ( N ( A)  N ( B))  ( N (B)  N (C))  ( N (C)  N (A))   khi đó
degG A  2  deg B B  2  deg G C  2  2n  3 suy ra
4
n
3
H  (VH ; EH )  G[V \{A; B}]

deg G A  1  deg B B  1 

Xét

đồ

thị


(bỏ

đi

2

đỉnh

A,

B)

ta



| VH | n  2

theo giả thiết quy nạp ta suy ra H chứa hai tam giác có

4
2
2
| EH | n  1  3 n  (n  1)  1
chung cạnh. Từ đó suy ra Gn  (V ; E ) luôn chứa hai tam giác có chung cạnh.

c) Giả sử kết luận c) đúng với n  1 hay Gn 1 chứa 2 tam giác có chung cạnh.
Xét đồ thị Gn  (V ; E ) , theo kết quả a) tồn tại 1 tam giác. Ta giả sử
Gn  (V ; E ) chứa tam giác ( A; B; C ) và deg G A  deg B B  degG C . Gọi t là số
tam giác của Gn  (V ; E ) .

Theo nguyên lý bù trừ ta có
| N ( A)  N ( B) |  | N (B)  N (C) |  | N (C)  N (A) |
| N(A) |  | N(B) |  | N(C) |  | N(A)  N(B)  N(C) |  | N(A)  N(B)  N(C) |
| N(A) |  | N(B) |  | N(C) | 2 n

Do đó

11


t | N ( A)  N ( B ) |  | N (B)  N (C) |  | N (C)  N (A) | 2 | N(A) |  | N(B) |  | N(C) | 2 n  2
Nếu | N(A) |  | N(B) |  | N(C) | 2 n  2  n | N(A) |  | N(B) |  | N(C) | 3n  2 ta có kết quả bài

toán.
Nếu | N(A) |  | N(B) |  | N(C) | 2 n  2  n | N(A) |  | N(B) |  | N(C) | 3n  2
2

Suy ra | N(A) |  | N(B) | 3 (3n  2)
Xét đồ thị H  (VH ; EH )  G[V \{A;B}]

(bỏ

đi

2

đỉnh

A,


B)

ta



| VH | n  2

theo giả thiết quy nạp ta suy ra H chứa n  1 tam

2
2
2
| EH | n  1  3 (3n  2)  1  (n  1)  1
giác. Từ đó suy ra Gn  (V ; E ) luôn chứa ít nhất n tam giác.

Kết quả a) của bài toán cho ta một đánh giá tương quan giữa số cạnh của đồ
thị với số đỉnh trong trường hợp đồ thị có chứa một tam giác. Tuy nhiên đây là đánh
giá chưa chặt vì kết quả cuối cùng là đồ thị có chứa n tam giác. Định lí sau cho ta
kết quả tổng quát trong trường hợp đồ thị không chứa tam giác.
Bài toán 8: (Định lí Mantel – Turan) Xét một đồ thị đơn vô hướng G  (V ; E ) có
 n2 
n đỉnh và k cạnh. Khi đó, nếu G  (V ; E ) không chứa tam giác thì k    .
4

Phân tích và lời giải.
Chứng minh quy nạp theo số đỉnh của đồ thị n .
Khi n  1, 2 hiển nhiên ta có kết quả bài toán. Giả sử bài toán đúng với mọi đồ
thị có it hơn n đỉnh.
Xét đồ thị G  (V , E ) có n đỉnh không chứa tam giác.

Lấy
sao cho
ta có
suy
A; B  V
( A; B)  E
N ( A)  N ( B)   ,
ra degG A  1  deg B B  1  n  2 .

Gọi H  (VH ; EH )  G[V \{A; B}] gồm n  2 đỉnh.
Khi đó, H không chứa tam giác nên theo giả thiết quy nạp ta có
| EH | [
12

(n  2) 2
] .
4


Do đó
 ( n  2) 2 
n2
| E | 

n

2

1


.

4
 4 
 n2 

Do | E | nguyên dương nên ta có | E |   .
4




Bài toán 9: (Rumani TST 2008) Một nhóm người được gọi là n- cân nếu nó
thoả mãn hai điều kiện:
i) Trong số ba người tuỳ ý, luôn tồn tại hai người quen nhau.
ii) Trong n người bất kì, luôn có ít nhất hai người không quen nhau.
Chứng minh rằng có nhiều nhất là

(n  1)(n  2)
người trong một nhóm n- cân.
2

Phân tích và lời giải.
Từ một nhóm người n-cân ta xây dựng đồ thị G  (V ; E ) trong đó mỗi người của
nhóm n- cân là một đỉnh, nếu hai người quen biết nhau ta có một cạnh của đồ thị. Ta
chứng minh | V |

(n  1)(n  2)
bằng phương pháp quy nạp theo n.
2


Khi n  2 thì khẳng định trên là hiển nhiên.
Giả sử kết quả đúng cho nhóm (n-1)- cân. Xét nhóm n-cân, ta xây dựng đồ thị
như trên.
Lấy A V , với mọi X , Y  M ( A) theo điều kiện i) thì phải có cạnh nối hai trong
ba đỉnh (A; X; Y) do đó ( X ; Y )  E . Điều này đúng với mọi X , Y  M ( A) nên theo điều
kiện ii) ta suy ra | M ( A) | n  1 .
Ta xét 2 trường hợp:
Trường hợp 1: | N ( A) | n  1 khi đó | V | 2n  1 

(n  1)(n  2)
.
2

Trường hơp 2: | N ( A) | n .
Xét tập con gồm n  1 đỉnh bất kì của N ( A) , thêm vào đó đỉnh A khi đó theo ii)
tồn tại X , Y sao cho ( X ; Y )  E . Rõ ràng X , Y thuộc tập con gồm n  1 đỉnh ban đầu.
Do đó N ( A) là một nhóm (n-1) – cân.
Theo giả thiết quy nạp ta có
| N ( A) |

(n  2)(n  1)
2

Suy ra | V | 1  n  1 

(n  2)(n  1) (n  1)(n  2)
.

2

2

Ta suy ra kết quả bài toán.
Bài tập tự luyện
Bài tập 1: (MOSP, 2011) Xét các số thực x1; x2 ;..; xn . Chứng minh rằng có không quá
 n2 
 4  cặp (i; j ) với 1  i  j  n sao cho 1 | xi  x j | 2 .
 

Bài tập 2: Nếu G là một đồ thị đơn có n đỉnh và không chứa tứ giác nào. Chứng
minh rằng số cạnh của G không vượt quá
13

1
n(1  4n  3) .
4


Bài tập 3: (USA TST 1995) Một đồ thị G gồm n đỉnh và k cạnh, G không chứa tam
giác. Chứng minh rằng chúng ta có thể chọn ra một đỉnh sao cho khi loại bỏ đỉnh đó
và các đỉnh kề của nó (cũng như các cạnh có 1 đầu mút nằm trong tập đỉnh này) ta
thu được đồ thị cảm sinh có nhiều nhất k (1 

4k
) cạnh.
n2

Bài tập 4: Cho n điểm P1; P2 ;..: Pn sao cho không có ba điểm nào thẳng hàng. Người ta
kẻ một số đoạn thẳng, mỗi đoạn nối 2 điểm. Biết rằng trong 4 điểm bất kì, luôn có ít
nhât một tam giác được kẻ. Hỏi đã có ít nhất bao nhiêu đoạn thẳng được kẻ.

Bài tập 5: (Ba Lan 1997). Chứng minh rằng nếu n điểm nằm trên một đường tròn
đơn vị thì có nhiều nhất là

n2
đoạn thẳng nối các cặp điểm trong chúng có độ dài lớn
3

hơn 2 .
Bài tập 6: Chứng minh rằng nếu đồ thì G có 2n+1 đỉnh và n 2  n  1 cạnh thì G chứa ít
nhất một tam giác.
Bài tập 7: (USA TST 2008) Với mỗi cặp điểm A( x1; y1 ) và B( x2 ; y2 ) trong mặt phẳng
(Oxy), ta đặt d ( A; B) | x1  x2 |  | y1  y2 |
Ta gọi cặp ( A; B) (không phân biệt thứ tự) là cặp điểm điều hoà nếu
1  d ( A; B)  2 . Xác định số cặp điểm điều hoà lớn nhất trong số 100 điểm cho
trước trong mặt phẳng.
Bài tập 8: Chứng minh rằng nếu đồ thị G gồm n đỉnh và k cạnh thì nó chứa ít nhất
k

3n

(4k  n 2 ) tam giác.

Bài tập 9: (VMO 2017) Cho số nguyên n  1 . Bảng ô vuông ABCD có kích thước
n  n gồm n 2 ô vuông đơn vị, mỗi ô vuông đơn vị được tô bởi một trong ba màu: đen,
trắng, xám. Một cách tô màu được gọi là đối xứng nếu mỗi ô có tâm trên đường chéo
AC được tô màu xám và mỗi cặp ô đối xứng qua AC được tô cùng màu đen hoặc
cùng màu trắng. Người ta điền vào mỗi ô xám số 0, mỗi ô trắng một số nguyên
dương và mỗi ô đen một số nguyên âm. Một cách điền số như vậy được gọi là k-cân
đối (với k nguyên dương) nếu thoả mãn hai điều kiện sau:
i) Mỗi cặp ô đối xứng qua AC được điền cùng một số nguyên thuộc [  k ; k ] .

ii) Nếu một hàng và một cột giao nhau tại ô đen thì tập các số nguyên dương
được điền trên hàng đó và tập các số nguyên dương được điền trên cột đó không giao
nhau; nếu một hàng và một cột giao nhau tại ô trắng thì tập các số nguyên âm được
điền trên hàng đó và tập các số nguyên âm được điền trên cột đó không giao nhau.
a) Với n  5 , tìm giá trị nhỏ nhất của k để tồn tại cách điền k- cân đối cho cách
tô màu đối xứng ở hình bên dưới.

14


b) Với n  2017 , tìm giá trị nhỏ nhất của k để với mọi cách tô màu đối xứng,
luôn tồn tại cách điền số k-cân đối.

15


TÀI LIỆU THAM KHẢO
[1]. Lời giải và bình luận đề thi VMO 2017, Trần Nam Dũng – Võ Quốc Bá
Cẩn – Trần Quang Hùng – Lê Phúc Lữ - Nguyễn Văn Huyện.
[2]. Extremal graph theory, David Conlon.
[3]. Bài giảng tổ hợp dành cho giáo viên 2013, Phan Thị Hà Dương – Viện toán
học.
Bài tập bổ sung:
Bài 6: Trong quốc gia VMO có nhiều hơn 100 thành phố và các đường bay( hai
chiều) nối một số các thành phố. Biết rằng thủ đô có đường bay trực tiếp đến 100
thành phố khác và một thành phố bất kì (không phải thủ đô) có đường bay trực tiếp
đến 10 thành phố khác, ngoài ra 2 thành phố bất kì đều được nối với nhau bằng một
đường bay trực tiếp hoặc gián tiếp. Chứng minh rằng có thể đóng một nửa số đường
bay phục vụ thủ đô sao cho vẫn có thể bay từ một thành phố bất kì đến một thành phố
bất kì khác.

Giải:
Gọi G là đồ thị trên tập đỉnh các thành phố, các cạnh tương ứng với các đường bay
trực tiếp. G’ là đồ thị cảm sinh bằng cách bỏ đi thủ đô.
Khi đó: G liên thông
G’ được phân hoạch thành các thành phần liên thông và mỗi thành phần phải chứa 1
đỉnh kề của T (thủ đô).
Do đó, mỗi thành phần liên thông chứa ít nhất 2 đỉnh bậc 9 và các đỉnh bậc 10.
Do mỗi đỉnh bậc 9 trong thành phần liên thông của G’ phải kề với T nên G’ có không
quá 50 thành phần liên thông (trong mỗi thành phần liên thông có 2 đỉnh nối với T) .
Ta có thể giữ lại tính liên thông bằng cách bỏ đi bớt đúng 01 cạnh nối T trong các
thành phần liên thông.
Bài 19: Nhà vua nước Nga quyết định xây dựng n thành phố và n-1 con đường liên kết giữa chúng:
mỗi con đường nối hai thành phố và không đi qua một thành phố khác; hai con đường bất kì không
giao nhau. Nhà vua cũng muốn rằng khoảng cách nhỏ nhất giữa hai thành phố (theo đường bộ) lần
n(n  1)
lượt là 1, 2, ..,
. Có thể thực hiện được yêu cầu của nhà vua không nếu:
2
a) n=6
b) n=2018
Hướng dẫn: a) xây dựng thông qua hình vẽ
b) mô hình hoá đồ thị, chứng minh phản chứng, chú ý tính chất số học của dãy khoảng cách.
Bài 20: Người ta điền vào mỗi ô của bảng vuông nxn một trong hai số 0 hoặc 1. Biết rằng với mỗi ô
chứa số 0 thì có ít nhất n ô nằm trên cùng hàng hoặc cùng cột với nó chứa số 1.
a) Tìm một cách điền khi n=5
b) Chứng minh rằng có ít nhất một nửa bảng được điền số 1 với n cho trước.
HD: mô hình hoá đồ thị để đánh giá số cạnh theo tính toán trực tiếp.
Chú ý chuyển điều kiện bài toán: thành các bất đẳng thức để đánh giá.

16




Tài liệu bạn tìm kiếm đã sẵn sàng tải về

Tải bản đầy đủ ngay
×